PT76.S2.Q19 vs PT24.S3.Q12 (Flaw)

lovejeb0729lovejeb0729 Alum Member
edited March 2023 in Logical Reasoning 123 karma

Hi

PT 76 S2 Q19 AC (D) is not a flaw because the author does not presume that the argument it cites is the union leaders' only argument for their view while PT 25 S2 Q12 AC (E) is a flaw because a reason not mentioned in the stimulus is not considered.

Both of the arguments do not contain phrases such as "the only reason why..." and yet they seem to give me contradictory responses. How do we know when premises are the only reasons why the author comes to the conclusion in an argument where there is no mention of the reasons being the only reasons?

Comments

  • KevinLuminateLSATKevinLuminateLSAT Alum Member
    edited March 2023 983 karma

    I think it's important to distinguish between the "view" and the "argument [the stimulus] cites" in answer choice (D).

    What is the "argument it cites"? Well, do you notice that PT76 S2 Q19's conclusion is "We should reject this argument."? That argument is in the first sentence, which describes something the union leaders "argue" - "increases in multinational control of manufacturing have shifted labor to nations without strong worker protections, resulting in a corresponding global decrease in workers' average wages."

    So if that's the "argument" cited by the stimulus, what is the "view" of the union leaders that this argument was supporting? It's mentioned in the second to last sentence - the union leaders want to "oppose multinational control".

    Now, look back at the conclusion. Did it say that we should reject the union leader's "view"? That we should not oppose multinational control? No - it just says that we should reject the argument described in the first sentence.

    This is why (D) is wrong. The author's conclusion was merely that we should reject one particular argument the union leaders gave in support of their view that we should oppose multinational control. But even if there were other arguments the union leaders had available in support of their view, the stimulus's conclusion had nothing to do with those arguments. The possibility of other arguments has no impact on whether the particular argument described in the stimulus should be rejected.

    I suspect you were interpreting the author's argument as something like this:

    Premise: The union leader's argument that legislators should oppose multinational control is based on their own self-interest.
    Conclusion: Thus, legislators should not oppose multinational control.

    But the argument was actually like this:

    Premise: The union leader's argument that legislators should oppose multinational control is based on their own self-interest.
    Conclusion: Thus, that argument is not persuasive and should not be considered.

    Under this interpretation, the author has no opinion on whether legislators should or should not oppose multinational control. The author might actually agree with the union leaders' ultimate view - but the author is trying to criticize the support that the leaders offered for that view.

Sign In or Register to comment.